免費論壇 繁體 | 簡體
Sclub交友聊天~加入聊天室當版主
分享
返回列表 发帖

一道反正切函数的级数问题

本帖最后由 青青子衿 于 2018-12-2 13:46 编辑

\[\color{black}{\sum\limits_{n=1}^{\infty}\arctan\frac{1}{n^2+1}=\arctan\left(\frac{\tan\left(\sqrt{\frac{\sqrt{2}-1}{2}}\pi\right)}{\tanh\left(\sqrt{\frac{\sqrt{2}+1}{2}}\pi\right)}\right)-\frac{\pi}{8}}\]
分享到: QQ空间QQ空间 腾讯微博腾讯微博 腾讯朋友腾讯朋友

回复 5# 战巡
谢谢站版主

TOP

返回列表 回复 发帖